A small doubt about the dominated convergence theorem The Next CEO of Stack OverflowIs Lebesgue's Dominated Convergence Theorem a logical equivalence?Lebesgue's Dominated Convergence Theorem questionsExample about Dominated Convergence TheoremDominated Convergence TheoremNecessity of generalization of Dominated Convergence theoremSeeking counterexample for Dominated Convergence theoremHypothesis of dominated convergence theoremDominated convergence theorem vs continuityBartle's proof of Lebesgue Dominated Convergence TheoremTheorem similar to dominated convergence theorem

Do I need to write [sic] when a number is less than 10 but isn't written out?

TikZ: How to reverse arrow direction without switching start/end point?

How to get from Geneva Airport to Metabief, Doubs, France by public transport?

Why does standard notation not preserve intervals (visually)

Does increasing your ability score affect your main stat?

Can we say or write : "No, it'sn't"?

How do I align (1) and (2)?

Where does this common spurious transmission come from? Is there a quality difference?

Solving system of ODEs with extra parameter

Is it professional to write unrelated content in an almost-empty email?

A Man With a Stainless Steel Endoskeleton (like The Terminator) Fighting Cloaked Aliens Only He Can See

Legal workarounds for testamentary trust perceived as unfair

How to edit “Name” property in GCI output?

If Nick Fury and Coulson already knew about aliens (Kree and Skrull) why did they wait until Thor's appearance to start making weapons?

Why did CATV standarize in 75 ohms and everyone else in 50?

Running a General Election and the European Elections together

Won the lottery - how do I keep the money?

Would a grinding machine be a simple and workable propulsion system for an interplanetary spacecraft?

Is there a difference between "Fahrstuhl" and "Aufzug"

Why isn't acceleration always zero whenever velocity is zero, such as the moment a ball bounces off a wall?

How is this set of matrices closed under multiplication?

Where do students learn to solve polynomial equations these days?

Can you be charged for obstruction for refusing to answer questions?

I want to delete every two lines after 3rd lines in file contain very large number of lines :



A small doubt about the dominated convergence theorem



The Next CEO of Stack OverflowIs Lebesgue's Dominated Convergence Theorem a logical equivalence?Lebesgue's Dominated Convergence Theorem questionsExample about Dominated Convergence TheoremDominated Convergence TheoremNecessity of generalization of Dominated Convergence theoremSeeking counterexample for Dominated Convergence theoremHypothesis of dominated convergence theoremDominated convergence theorem vs continuityBartle's proof of Lebesgue Dominated Convergence TheoremTheorem similar to dominated convergence theorem










3












$begingroup$



Theorem $mathbfA.2.11$ (Dominated convergence). Let $f_n : X to mathbb R$ be a sequence of measurable functions and assume that there exists some integrable function $g : X to mathbb R$ such that $|f_n(x)| leq |g(x)|$ for $mu$-almost every $x$ in $X$. Assume moreover that the sequence $(f_n)_n$ converges at $mu$-almost every point to some function $f : X to mathbb R$. Then $f$ is integrable and satisfies $$lim_n int f_n , dmu = int f , dmu.$$




I wanted to know if in the hypothesis $|f_n(x)| leq|g(x)|$ above, if I already know that each $f_n$ is integrable, besides convergent, the theorem remains valid? Without me having to find this $g$ integrable?










share|cite|improve this question











$endgroup$











  • $begingroup$
    The dominating function $g$ is the whole point of the theorem. It's in the very name of the theorem.
    $endgroup$
    – Nate Eldredge
    9 mins ago















3












$begingroup$



Theorem $mathbfA.2.11$ (Dominated convergence). Let $f_n : X to mathbb R$ be a sequence of measurable functions and assume that there exists some integrable function $g : X to mathbb R$ such that $|f_n(x)| leq |g(x)|$ for $mu$-almost every $x$ in $X$. Assume moreover that the sequence $(f_n)_n$ converges at $mu$-almost every point to some function $f : X to mathbb R$. Then $f$ is integrable and satisfies $$lim_n int f_n , dmu = int f , dmu.$$




I wanted to know if in the hypothesis $|f_n(x)| leq|g(x)|$ above, if I already know that each $f_n$ is integrable, besides convergent, the theorem remains valid? Without me having to find this $g$ integrable?










share|cite|improve this question











$endgroup$











  • $begingroup$
    The dominating function $g$ is the whole point of the theorem. It's in the very name of the theorem.
    $endgroup$
    – Nate Eldredge
    9 mins ago













3












3








3


1



$begingroup$



Theorem $mathbfA.2.11$ (Dominated convergence). Let $f_n : X to mathbb R$ be a sequence of measurable functions and assume that there exists some integrable function $g : X to mathbb R$ such that $|f_n(x)| leq |g(x)|$ for $mu$-almost every $x$ in $X$. Assume moreover that the sequence $(f_n)_n$ converges at $mu$-almost every point to some function $f : X to mathbb R$. Then $f$ is integrable and satisfies $$lim_n int f_n , dmu = int f , dmu.$$




I wanted to know if in the hypothesis $|f_n(x)| leq|g(x)|$ above, if I already know that each $f_n$ is integrable, besides convergent, the theorem remains valid? Without me having to find this $g$ integrable?










share|cite|improve this question











$endgroup$





Theorem $mathbfA.2.11$ (Dominated convergence). Let $f_n : X to mathbb R$ be a sequence of measurable functions and assume that there exists some integrable function $g : X to mathbb R$ such that $|f_n(x)| leq |g(x)|$ for $mu$-almost every $x$ in $X$. Assume moreover that the sequence $(f_n)_n$ converges at $mu$-almost every point to some function $f : X to mathbb R$. Then $f$ is integrable and satisfies $$lim_n int f_n , dmu = int f , dmu.$$




I wanted to know if in the hypothesis $|f_n(x)| leq|g(x)|$ above, if I already know that each $f_n$ is integrable, besides convergent, the theorem remains valid? Without me having to find this $g$ integrable?







measure-theory convergence lebesgue-integral






share|cite|improve this question















share|cite|improve this question













share|cite|improve this question




share|cite|improve this question








edited 2 hours ago









Rócherz

3,0013821




3,0013821










asked 2 hours ago









Ricardo FreireRicardo Freire

579211




579211











  • $begingroup$
    The dominating function $g$ is the whole point of the theorem. It's in the very name of the theorem.
    $endgroup$
    – Nate Eldredge
    9 mins ago
















  • $begingroup$
    The dominating function $g$ is the whole point of the theorem. It's in the very name of the theorem.
    $endgroup$
    – Nate Eldredge
    9 mins ago















$begingroup$
The dominating function $g$ is the whole point of the theorem. It's in the very name of the theorem.
$endgroup$
– Nate Eldredge
9 mins ago




$begingroup$
The dominating function $g$ is the whole point of the theorem. It's in the very name of the theorem.
$endgroup$
– Nate Eldredge
9 mins ago










2 Answers
2






active

oldest

votes


















3












$begingroup$

This is an excellent question. For the theorem to apply, you need the $f_n$'s to be uniformly dominated by an integrable function $g$. To see this, consider the sequence
$$
f_n(x) := frac1n mathbf1_[0,n](x).
$$

Clearly, $f_n in L^1(mathbbR)$ for each $n in mathbbN$. Moreover, $f_n(x) to 0$ as $n to infty$ for each $x in mathbbR$. However,
beginalign*
lim_n to infty int_mathbbR f_n,mathrmdm = lim_n to infty int_0^n frac1n,mathrmdx = 1 neq 0.
endalign*



Nevertheless, you are not in too much trouble if you cannot find a dominating function. If your sequence of functions is uniformly bounded in $L^p(E)$ for $1 < p < infty$ where $E$ has finite measure, then you can still take the limit inside the integral. Namely, the following theorem often helps to rectify the situation.




Theorem. Let $(f_n)$ be a sequence of measurable functions on a measure space $(X,mathfrakM,mu)$ converging almost everywhere to a measurable function $f$. If $E subset X$ has finite measure and $(f_n)$ is bounded in $L^p(E)$ for some $1 < p < infty$, then
$$
lim_n to infty int_E f_n,mathrmdmu = int_E f,mathrmdmu.
$$

In fact, one has $f_n to f$ strongly in $L^1(E)$.




In a sense, one can do without a dominating function when the sequence is uniformly bounded in a "higher $L^p$-space" and the domain of integration has finite measure.






share|cite|improve this answer











$endgroup$












  • $begingroup$
    I understood. Thanks a lot for the help
    $endgroup$
    – Ricardo Freire
    1 hour ago


















2












$begingroup$

In general, it is not sufficient that each $f_n$ be integrable without a dominating function. For instance, the functions $f_n = chi_[n,n+1]$ on $mathbf R_ge 0$ are all integrable, and $f_n(x) to 0$ for all $xin mathbf R_ge 0$, but they are not dominated by an integrable function $g$, and indeed we do not have
$$
lim_ntoinfty int f_n = int lim_ntoinftyf_n
$$

since in this case, the left-hand side is $1$, but the right-hand side is $0$.




To see why there is no dominating function $g$, such a function would have the property that $g(x)ge 1$ for each $xge 0$, so it would not be integrable on $mathbf R_ge 0$.






share|cite|improve this answer









$endgroup$












  • $begingroup$
    I understood. Thanks a lot for the help
    $endgroup$
    – Ricardo Freire
    1 hour ago











Your Answer





StackExchange.ifUsing("editor", function ()
return StackExchange.using("mathjaxEditing", function ()
StackExchange.MarkdownEditor.creationCallbacks.add(function (editor, postfix)
StackExchange.mathjaxEditing.prepareWmdForMathJax(editor, postfix, [["$", "$"], ["\\(","\\)"]]);
);
);
, "mathjax-editing");

StackExchange.ready(function()
var channelOptions =
tags: "".split(" "),
id: "69"
;
initTagRenderer("".split(" "), "".split(" "), channelOptions);

StackExchange.using("externalEditor", function()
// Have to fire editor after snippets, if snippets enabled
if (StackExchange.settings.snippets.snippetsEnabled)
StackExchange.using("snippets", function()
createEditor();
);

else
createEditor();

);

function createEditor()
StackExchange.prepareEditor(
heartbeatType: 'answer',
autoActivateHeartbeat: false,
convertImagesToLinks: true,
noModals: true,
showLowRepImageUploadWarning: true,
reputationToPostImages: 10,
bindNavPrevention: true,
postfix: "",
imageUploader:
brandingHtml: "Powered by u003ca class="icon-imgur-white" href="https://imgur.com/"u003eu003c/au003e",
contentPolicyHtml: "User contributions licensed under u003ca href="https://creativecommons.org/licenses/by-sa/3.0/"u003ecc by-sa 3.0 with attribution requiredu003c/au003e u003ca href="https://stackoverflow.com/legal/content-policy"u003e(content policy)u003c/au003e",
allowUrls: true
,
noCode: true, onDemand: true,
discardSelector: ".discard-answer"
,immediatelyShowMarkdownHelp:true
);



);













draft saved

draft discarded


















StackExchange.ready(
function ()
StackExchange.openid.initPostLogin('.new-post-login', 'https%3a%2f%2fmath.stackexchange.com%2fquestions%2f3168945%2fa-small-doubt-about-the-dominated-convergence-theorem%23new-answer', 'question_page');

);

Post as a guest















Required, but never shown

























2 Answers
2






active

oldest

votes








2 Answers
2






active

oldest

votes









active

oldest

votes






active

oldest

votes









3












$begingroup$

This is an excellent question. For the theorem to apply, you need the $f_n$'s to be uniformly dominated by an integrable function $g$. To see this, consider the sequence
$$
f_n(x) := frac1n mathbf1_[0,n](x).
$$

Clearly, $f_n in L^1(mathbbR)$ for each $n in mathbbN$. Moreover, $f_n(x) to 0$ as $n to infty$ for each $x in mathbbR$. However,
beginalign*
lim_n to infty int_mathbbR f_n,mathrmdm = lim_n to infty int_0^n frac1n,mathrmdx = 1 neq 0.
endalign*



Nevertheless, you are not in too much trouble if you cannot find a dominating function. If your sequence of functions is uniformly bounded in $L^p(E)$ for $1 < p < infty$ where $E$ has finite measure, then you can still take the limit inside the integral. Namely, the following theorem often helps to rectify the situation.




Theorem. Let $(f_n)$ be a sequence of measurable functions on a measure space $(X,mathfrakM,mu)$ converging almost everywhere to a measurable function $f$. If $E subset X$ has finite measure and $(f_n)$ is bounded in $L^p(E)$ for some $1 < p < infty$, then
$$
lim_n to infty int_E f_n,mathrmdmu = int_E f,mathrmdmu.
$$

In fact, one has $f_n to f$ strongly in $L^1(E)$.




In a sense, one can do without a dominating function when the sequence is uniformly bounded in a "higher $L^p$-space" and the domain of integration has finite measure.






share|cite|improve this answer











$endgroup$












  • $begingroup$
    I understood. Thanks a lot for the help
    $endgroup$
    – Ricardo Freire
    1 hour ago















3












$begingroup$

This is an excellent question. For the theorem to apply, you need the $f_n$'s to be uniformly dominated by an integrable function $g$. To see this, consider the sequence
$$
f_n(x) := frac1n mathbf1_[0,n](x).
$$

Clearly, $f_n in L^1(mathbbR)$ for each $n in mathbbN$. Moreover, $f_n(x) to 0$ as $n to infty$ for each $x in mathbbR$. However,
beginalign*
lim_n to infty int_mathbbR f_n,mathrmdm = lim_n to infty int_0^n frac1n,mathrmdx = 1 neq 0.
endalign*



Nevertheless, you are not in too much trouble if you cannot find a dominating function. If your sequence of functions is uniformly bounded in $L^p(E)$ for $1 < p < infty$ where $E$ has finite measure, then you can still take the limit inside the integral. Namely, the following theorem often helps to rectify the situation.




Theorem. Let $(f_n)$ be a sequence of measurable functions on a measure space $(X,mathfrakM,mu)$ converging almost everywhere to a measurable function $f$. If $E subset X$ has finite measure and $(f_n)$ is bounded in $L^p(E)$ for some $1 < p < infty$, then
$$
lim_n to infty int_E f_n,mathrmdmu = int_E f,mathrmdmu.
$$

In fact, one has $f_n to f$ strongly in $L^1(E)$.




In a sense, one can do without a dominating function when the sequence is uniformly bounded in a "higher $L^p$-space" and the domain of integration has finite measure.






share|cite|improve this answer











$endgroup$












  • $begingroup$
    I understood. Thanks a lot for the help
    $endgroup$
    – Ricardo Freire
    1 hour ago













3












3








3





$begingroup$

This is an excellent question. For the theorem to apply, you need the $f_n$'s to be uniformly dominated by an integrable function $g$. To see this, consider the sequence
$$
f_n(x) := frac1n mathbf1_[0,n](x).
$$

Clearly, $f_n in L^1(mathbbR)$ for each $n in mathbbN$. Moreover, $f_n(x) to 0$ as $n to infty$ for each $x in mathbbR$. However,
beginalign*
lim_n to infty int_mathbbR f_n,mathrmdm = lim_n to infty int_0^n frac1n,mathrmdx = 1 neq 0.
endalign*



Nevertheless, you are not in too much trouble if you cannot find a dominating function. If your sequence of functions is uniformly bounded in $L^p(E)$ for $1 < p < infty$ where $E$ has finite measure, then you can still take the limit inside the integral. Namely, the following theorem often helps to rectify the situation.




Theorem. Let $(f_n)$ be a sequence of measurable functions on a measure space $(X,mathfrakM,mu)$ converging almost everywhere to a measurable function $f$. If $E subset X$ has finite measure and $(f_n)$ is bounded in $L^p(E)$ for some $1 < p < infty$, then
$$
lim_n to infty int_E f_n,mathrmdmu = int_E f,mathrmdmu.
$$

In fact, one has $f_n to f$ strongly in $L^1(E)$.




In a sense, one can do without a dominating function when the sequence is uniformly bounded in a "higher $L^p$-space" and the domain of integration has finite measure.






share|cite|improve this answer











$endgroup$



This is an excellent question. For the theorem to apply, you need the $f_n$'s to be uniformly dominated by an integrable function $g$. To see this, consider the sequence
$$
f_n(x) := frac1n mathbf1_[0,n](x).
$$

Clearly, $f_n in L^1(mathbbR)$ for each $n in mathbbN$. Moreover, $f_n(x) to 0$ as $n to infty$ for each $x in mathbbR$. However,
beginalign*
lim_n to infty int_mathbbR f_n,mathrmdm = lim_n to infty int_0^n frac1n,mathrmdx = 1 neq 0.
endalign*



Nevertheless, you are not in too much trouble if you cannot find a dominating function. If your sequence of functions is uniformly bounded in $L^p(E)$ for $1 < p < infty$ where $E$ has finite measure, then you can still take the limit inside the integral. Namely, the following theorem often helps to rectify the situation.




Theorem. Let $(f_n)$ be a sequence of measurable functions on a measure space $(X,mathfrakM,mu)$ converging almost everywhere to a measurable function $f$. If $E subset X$ has finite measure and $(f_n)$ is bounded in $L^p(E)$ for some $1 < p < infty$, then
$$
lim_n to infty int_E f_n,mathrmdmu = int_E f,mathrmdmu.
$$

In fact, one has $f_n to f$ strongly in $L^1(E)$.




In a sense, one can do without a dominating function when the sequence is uniformly bounded in a "higher $L^p$-space" and the domain of integration has finite measure.







share|cite|improve this answer














share|cite|improve this answer



share|cite|improve this answer








edited 1 hour ago

























answered 2 hours ago









rolandcyprolandcyp

1,856315




1,856315











  • $begingroup$
    I understood. Thanks a lot for the help
    $endgroup$
    – Ricardo Freire
    1 hour ago
















  • $begingroup$
    I understood. Thanks a lot for the help
    $endgroup$
    – Ricardo Freire
    1 hour ago















$begingroup$
I understood. Thanks a lot for the help
$endgroup$
– Ricardo Freire
1 hour ago




$begingroup$
I understood. Thanks a lot for the help
$endgroup$
– Ricardo Freire
1 hour ago











2












$begingroup$

In general, it is not sufficient that each $f_n$ be integrable without a dominating function. For instance, the functions $f_n = chi_[n,n+1]$ on $mathbf R_ge 0$ are all integrable, and $f_n(x) to 0$ for all $xin mathbf R_ge 0$, but they are not dominated by an integrable function $g$, and indeed we do not have
$$
lim_ntoinfty int f_n = int lim_ntoinftyf_n
$$

since in this case, the left-hand side is $1$, but the right-hand side is $0$.




To see why there is no dominating function $g$, such a function would have the property that $g(x)ge 1$ for each $xge 0$, so it would not be integrable on $mathbf R_ge 0$.






share|cite|improve this answer









$endgroup$












  • $begingroup$
    I understood. Thanks a lot for the help
    $endgroup$
    – Ricardo Freire
    1 hour ago















2












$begingroup$

In general, it is not sufficient that each $f_n$ be integrable without a dominating function. For instance, the functions $f_n = chi_[n,n+1]$ on $mathbf R_ge 0$ are all integrable, and $f_n(x) to 0$ for all $xin mathbf R_ge 0$, but they are not dominated by an integrable function $g$, and indeed we do not have
$$
lim_ntoinfty int f_n = int lim_ntoinftyf_n
$$

since in this case, the left-hand side is $1$, but the right-hand side is $0$.




To see why there is no dominating function $g$, such a function would have the property that $g(x)ge 1$ for each $xge 0$, so it would not be integrable on $mathbf R_ge 0$.






share|cite|improve this answer









$endgroup$












  • $begingroup$
    I understood. Thanks a lot for the help
    $endgroup$
    – Ricardo Freire
    1 hour ago













2












2








2





$begingroup$

In general, it is not sufficient that each $f_n$ be integrable without a dominating function. For instance, the functions $f_n = chi_[n,n+1]$ on $mathbf R_ge 0$ are all integrable, and $f_n(x) to 0$ for all $xin mathbf R_ge 0$, but they are not dominated by an integrable function $g$, and indeed we do not have
$$
lim_ntoinfty int f_n = int lim_ntoinftyf_n
$$

since in this case, the left-hand side is $1$, but the right-hand side is $0$.




To see why there is no dominating function $g$, such a function would have the property that $g(x)ge 1$ for each $xge 0$, so it would not be integrable on $mathbf R_ge 0$.






share|cite|improve this answer









$endgroup$



In general, it is not sufficient that each $f_n$ be integrable without a dominating function. For instance, the functions $f_n = chi_[n,n+1]$ on $mathbf R_ge 0$ are all integrable, and $f_n(x) to 0$ for all $xin mathbf R_ge 0$, but they are not dominated by an integrable function $g$, and indeed we do not have
$$
lim_ntoinfty int f_n = int lim_ntoinftyf_n
$$

since in this case, the left-hand side is $1$, but the right-hand side is $0$.




To see why there is no dominating function $g$, such a function would have the property that $g(x)ge 1$ for each $xge 0$, so it would not be integrable on $mathbf R_ge 0$.







share|cite|improve this answer












share|cite|improve this answer



share|cite|improve this answer










answered 2 hours ago









Alex OrtizAlex Ortiz

11.2k21441




11.2k21441











  • $begingroup$
    I understood. Thanks a lot for the help
    $endgroup$
    – Ricardo Freire
    1 hour ago
















  • $begingroup$
    I understood. Thanks a lot for the help
    $endgroup$
    – Ricardo Freire
    1 hour ago















$begingroup$
I understood. Thanks a lot for the help
$endgroup$
– Ricardo Freire
1 hour ago




$begingroup$
I understood. Thanks a lot for the help
$endgroup$
– Ricardo Freire
1 hour ago

















draft saved

draft discarded
















































Thanks for contributing an answer to Mathematics Stack Exchange!


  • Please be sure to answer the question. Provide details and share your research!

But avoid


  • Asking for help, clarification, or responding to other answers.

  • Making statements based on opinion; back them up with references or personal experience.

Use MathJax to format equations. MathJax reference.


To learn more, see our tips on writing great answers.




draft saved


draft discarded














StackExchange.ready(
function ()
StackExchange.openid.initPostLogin('.new-post-login', 'https%3a%2f%2fmath.stackexchange.com%2fquestions%2f3168945%2fa-small-doubt-about-the-dominated-convergence-theorem%23new-answer', 'question_page');

);

Post as a guest















Required, but never shown





















































Required, but never shown














Required, but never shown












Required, but never shown







Required, but never shown

































Required, but never shown














Required, but never shown












Required, but never shown







Required, but never shown







Popular posts from this blog

Log på Navigationsmenu

Wonderful Copenhagen (sang) Eksterne henvisninger | NavigationsmenurSide på frankloesser.comWonderful Copenhagen

Detroit Tigers Spis treści Historia | Skład zespołu | Sukcesy | Członkowie Baseball Hall of Fame | Zastrzeżone numery | Przypisy | Menu nawigacyjneEncyclopedia of Detroit - Detroit TigersTigers Stadium, Detroit, MITigers Timeline 1900sDetroit Tigers Team History & EncyclopediaTigers Timeline 1910s1935 World Series1945 World Series1945 World Series1984 World SeriesComerica Park, Detroit, MI2006 World Series2012 World SeriesDetroit Tigers 40-Man RosterDetroit Tigers Coaching StaffTigers Hall of FamersTigers Retired Numberse